Liczby calkowite

Podzielność. Reszty z dzielenia. Kongruencje. Systemy pozycyjne. Równania diofantyczne. Liczby pierwsze i względnie pierwsze. NWW i NWD.
olgalagowska
Użytkownik
Użytkownik
Posty: 88
Rejestracja: 28 paź 2010, o 13:05
Płeć: Kobieta
Podziękował: 8 razy

Liczby calkowite

Post autor: olgalagowska »

Udowodnij, ze jesli liczba \(\displaystyle{ a+\frac{1}{a}}\), jest liczba calkowita, to rowniez \(\displaystyle{ a^k+\frac{1}{a^k}}\) jest liczba calkowita.
Użytkownik
Użytkownik
Posty: 9833
Rejestracja: 18 gru 2007, o 03:54
Płeć: Mężczyzna
Lokalizacja: Bydgoszcz
Podziękował: 90 razy
Pomógł: 2632 razy

Liczby calkowite

Post autor: »

Wskazówka: indukcja matematyczna.

Q.
Pacek
Użytkownik
Użytkownik
Posty: 26
Rejestracja: 9 sie 2010, o 14:36
Płeć: Kobieta
Lokalizacja: Warszawa
Pomógł: 5 razy

Liczby calkowite

Post autor: Pacek »

Skorzystamy z indukcji matematycznej.
Dla k=1 założenie zadania
Dla k=2
\(\displaystyle{ a ^{2} + \frac{1}{a^{2}} = \left( a+ \frac{1}{a} \right) ^{2} - 2 \cdot a \cdot \frac{1}{a}}\)
Skoros \(\displaystyle{ a+\frac{1}{a}}\) jest liczbą całkowitą to
\(\displaystyle{ \left( a+ \frac{1}{a} \right) ^{2}}\) teź jest liczą całkowitą.
Liczba dwa jest liczbą całkowitą. Liczba całkowita minus liczba całkowita to liczba całowita.
Zatem \(\displaystyle{ a^{2} + \frac{1}{a^{2}}}\) jest liczbą całkowitą.

Teraz zakładamy, że dla n=2,3, ... k-1 \(\displaystyle{ a ^{n} + \frac{1}{a^{n}}}\) jest liczbą całkowitą.
Musimy pokazać, że \(\displaystyle{ a ^{k} + \frac{1}{a^{k}}}\) też jest liczbą całkowitą.
Aby to udowdnić trzeba \(\displaystyle{ a ^{k} + \frac{1}{a^{k}}}\) rozpisać analogicznie jak dla 2 korzystając ze wzoru skróconego mnożenia:
\(\displaystyle{ a ^{k} + \frac{1}{a^{k}} = \left( a+ \frac{1}{a} \right) ^{k}- {n \choose 1}a^{k-1}\frac{1}{a}-
{n \choose 2}a^{k-2}\frac{1}{a^{2}}-}\)
...itd
Ostatnio zmieniony 8 paź 2011, o 12:19 przez Lbubsazob, łącznie zmieniany 1 raz.
Powód: Poprawa wiadomości - skalowanie nawiasów.
Użytkownik
Użytkownik
Posty: 9833
Rejestracja: 18 gru 2007, o 03:54
Płeć: Mężczyzna
Lokalizacja: Bydgoszcz
Podziękował: 90 razy
Pomógł: 2632 razy

Liczby calkowite

Post autor: »

Nie no, można o wiele prościej, korzystając z tego, że:
\(\displaystyle{ \left( a+\frac 1a\right) \left( a^k+\frac{1}{a^k}\right) =a^{k+1}+\frac{1}{a^{k+1}}+a^{k-1}+\frac{1}{a^{k-1}}}\)

Q.
olgalagowska
Użytkownik
Użytkownik
Posty: 88
Rejestracja: 28 paź 2010, o 13:05
Płeć: Kobieta
Podziękował: 8 razy

Liczby calkowite

Post autor: olgalagowska »

Qń pisze:Nie no, można o wiele prościej, korzystając z tego, że:
\(\displaystyle{ \left( a+\frac 1a\right) \left( a^k+\frac{1}{a^k}\right) =a^{k+1}+\frac{1}{a^{k+1}}+a^{k-1}+\frac{1}{a^{k-1}}}\)

Q.
Korzystajac z tego, jak mozna wyjasnic ze to bedzie liczba calkowita?
Użytkownik
Użytkownik
Posty: 9833
Rejestracja: 18 gru 2007, o 03:54
Płeć: Mężczyzna
Lokalizacja: Bydgoszcz
Podziękował: 90 razy
Pomógł: 2632 razy

Liczby calkowite

Post autor: »

Qń pisze:Wskazówka: indukcja matematyczna.
Sprawdź prawdziwość tezy dla \(\displaystyle{ k=1,2}\) (podstawa indukcji), a następnie załóż, że twierdzenie jest prawdziwe dla \(\displaystyle{ k-1,k}\) i wywnioskuj z tego prawdziwość dla \(\displaystyle{ k+1}\) (krok indukcyjny).

Q.
major37
Użytkownik
Użytkownik
Posty: 1631
Rejestracja: 30 wrz 2010, o 13:56
Płeć: Mężczyzna
Lokalizacja: Witaszyce
Podziękował: 288 razy
Pomógł: 72 razy

Liczby calkowite

Post autor: major37 »

Ale tu nie mamy udowodnić dla liczb naturalnych a indukcja chyba tego dotyczy ?
Użytkownik
Użytkownik
Posty: 9833
Rejestracja: 18 gru 2007, o 03:54
Płeć: Mężczyzna
Lokalizacja: Bydgoszcz
Podziękował: 90 razy
Pomógł: 2632 razy

Liczby calkowite

Post autor: »

\(\displaystyle{ k}\) jest przecież naturalne.

Q.
ODPOWIEDZ